How can I solve this integral involving cosine and sine functions?

  • Thread starter Thread starter lukatwo
  • Start date Start date
  • Tags Tags
    Integral
Click For Summary
The integral ∫(cos^3(x)/√(sin(x)))dx is approached by substituting sin(x) with t, leading to the transformation of the integral into a simpler form. The user breaks down the integral into two parts: ∫(1/√t)dt and ∫(t^2/√t)dt, intending to solve them separately. However, Wolfram suggests an alternative substitution method, which the user finds unnecessary since both methods yield similar results. The discussion highlights the validity of different approaches to solving the integral, ultimately confirming that both methods are correct. The conversation emphasizes the importance of understanding various techniques in integral calculus.
lukatwo
Messages
24
Reaction score
0

Homework Statement



I've been trying to integrate the following: ∫[STRIKE]\frac{cos^3(x)}{\sqrt{sin(x)}}[/STRIKE]dx

Homework Equations


The Attempt at a Solution



First, I substituted sin(x) with t, and got dt=cos(x)dx => dx=\frac{dt}{cos(x)}.
After that I got ∫[STRIKE]\frac{cos^2(x)}{\sqrt{t}}[/STRIKE]dt
Then i transformed cos^2(x) into 1-sin^2(x), and finally got to ∫[STRIKE]\frac{1-t^2}{\sqrt{t}}[/STRIKE]dt

I thought I could just disintegrate them into two smaller integrals like ∫[STRIKE]\frac{1}{\sqrt{t}}[/STRIKE]dt - ∫[STRIKE]\frac{t^2}{\sqrt{t}}[/STRIKE]dt , and solve them easily, and then reverse the substitution.

Wolfram proposes that i cannot(?) do that, or rather prefers that I do another substitution.

I even tried to make it a defined integral, and calculate the values between the Wolfram solution, and my own. They differ by 0.1 or something similar.

Can someone explain what is the right way to do it?
 
Physics news on Phys.org
Both are correct. Wolfram just does another substitution which isn't really too necessary. Both give the same correct integral.
 
Thank you very much for your help!
 
Question: A clock's minute hand has length 4 and its hour hand has length 3. What is the distance between the tips at the moment when it is increasing most rapidly?(Putnam Exam Question) Answer: Making assumption that both the hands moves at constant angular velocities, the answer is ## \sqrt{7} .## But don't you think this assumption is somewhat doubtful and wrong?

Similar threads

  • · Replies 105 ·
4
Replies
105
Views
6K
  • · Replies 22 ·
Replies
22
Views
3K
  • · Replies 5 ·
Replies
5
Views
2K
  • · Replies 3 ·
Replies
3
Views
2K
  • · Replies 15 ·
Replies
15
Views
2K
  • · Replies 8 ·
Replies
8
Views
2K
  • · Replies 9 ·
Replies
9
Views
1K
  • · Replies 23 ·
Replies
23
Views
2K
Replies
9
Views
2K
  • · Replies 11 ·
Replies
11
Views
3K